Partial derivative second order

Click For Summary
The discussion revolves around solving a problem involving the second-order partial derivative of a function. The user initially expresses confusion about how to approach the question, particularly regarding the application of the quotient rule. After working through the calculations, they arrive at the correct solution, which is confirmed by others in the thread. There is also a mention of personal preference in using different methods, such as the chain rule, to achieve the same result. The conversation concludes with a note on formatting etiquette in the forum.
Taylor_1989
Messages
400
Reaction score
14

Homework Statement


Hi guys, I am have a problem with the question displayed below:

upload_2017-2-15_23-33-6.png
[/B]

Its 6.1 ii) I am really not sure how I am suppose to approach this. I am new to partials, so any advice would be great.

Homework Equations

The Attempt at a Solution


So far I have:
$$\frac{\partial ^2 f}{\partial x^2}=\frac{\partial}{\partial x}\frac{\partial f}{\partial x}=\frac{\partial}{\partial x}\frac{-x}{r^3} $$

using quotient rule:

$$=[\frac{\partial}{\partial x}(-x)(r^3)-(-x)(\frac{\partial}{\partial x}r^3)]/r^6$$

$$\frac{\partial}{\partial x}r^3=3r^2\frac{\partial r}{\partial x}=3r^2*(x/r)=3rx$$

subbing the above into the quotient and simplifying
I get
$$\frac{-r^2+3x^2}{r^5}$$

[/B]
 
Physics news on Phys.org
Taylor_1989 said:
using quotient rule:
might not be the best thing to do. How did you find ##\partial f\over\partial x## ?
And: was 6 i) all right and clear ? How come 6 ii) is then problematic ?

[edit] Stupid me o:) . Your working is completely correct. Well done...
'Quotient rule' confused me, but it works just fine. My approach would be to use the chain rule -- with, of course, the same result. Matter of preference, not matter of 'best thing to do'.
 
Last edited:
Taylor_1989 said:

Homework Statement


Hi guys, I am have a problem with the question displayed below:

View attachment 113277[/B]

Its 6.1 ii) I am really not sure how I am suppose to approach this. I am new to partials, so any advice would be great.

Homework Equations

The Attempt at a Solution


So far I have:
$$\frac{\partial ^2 f}{\partial x^2}=\frac{\partial}{\partial x}\frac{\partial f}{\partial x}=\frac{\partial}{\partial x}\frac{-x}{r^3} $$

using quotient rule:

$$=[\frac{\partial}{\partial x}(-x)(r^3)-(-x)(\frac{\partial}{\partial x}r^3)]/r^6$$

$$\frac{\partial}{\partial x}r^3=3r^2\frac{\partial r}{\partial x}=3r^2*(x/r)=3rx$$

subbing the above into the quotient and simplifying
I get
$$\frac{-r^2+3x^2}{r^5}$$
[/B]

Your answer is correct. It is a matter of taste whether you leave your final numerator as ##3x^2-r^2##, or re-write it as ##2x^2-y^2-z^2##.

BTW: Remove all those offensive bold fonts: it looks like you are yelling at us.
 
thank for the responses. I will remove bold next, i did not even notice it was bold.
 
Question: A clock's minute hand has length 4 and its hour hand has length 3. What is the distance between the tips at the moment when it is increasing most rapidly?(Putnam Exam Question) Answer: Making assumption that both the hands moves at constant angular velocities, the answer is ## \sqrt{7} .## But don't you think this assumption is somewhat doubtful and wrong?

Similar threads

  • · Replies 4 ·
Replies
4
Views
1K
  • · Replies 3 ·
Replies
3
Views
3K
  • · Replies 5 ·
Replies
5
Views
1K
Replies
3
Views
2K
Replies
4
Views
1K
  • · Replies 1 ·
Replies
1
Views
1K
  • · Replies 6 ·
Replies
6
Views
2K
  • · Replies 7 ·
Replies
7
Views
2K
Replies
6
Views
1K
  • · Replies 6 ·
Replies
6
Views
2K